Difference between revisions of "2003 AMC 8 Problems/Problem 14"
(→Solution) |
(→Video Solution) |
||
(2 intermediate revisions by the same user not shown) | |||
Line 10: | Line 10: | ||
==Solution== | ==Solution== | ||
Since both T's are 7, then O has to equal 4, because 7 + 7 = 14. Then, F has to equal 1. To get R, we do 4 + 4 (since O = 4) to get R = 8. The value for W then has to be a number less than 5, otherwise it will change the value of O, and can't be a number that has already been used, like 4 or 1. The only other possibilities are 2 and 3. 2 doesn't work because it makes U = 4, which is what O already equals. So, the only possible value of W is 3 <math>\boxed{\text{D}}</math> | Since both T's are 7, then O has to equal 4, because 7 + 7 = 14. Then, F has to equal 1. To get R, we do 4 + 4 (since O = 4) to get R = 8. The value for W then has to be a number less than 5, otherwise it will change the value of O, and can't be a number that has already been used, like 4 or 1. The only other possibilities are 2 and 3. 2 doesn't work because it makes U = 4, which is what O already equals. So, the only possible value of W is 3 <math>\boxed{\text{D}}</math> | ||
− | |||
==Video Solution== | ==Video Solution== | ||
+ | https://www.youtube.com/watch?v=372B3Hyt3bA | ||
− | + | ~David | |
==See Also== | ==See Also== | ||
{{AMC8 box|year=2003|num-b=13|num-a=15}} | {{AMC8 box|year=2003|num-b=13|num-a=15}} | ||
{{MAA Notice}} | {{MAA Notice}} |
Latest revision as of 18:56, 15 April 2023
Contents
Problem
In this addition problem, each letter stands for a different digit.
If T = 7 and the letter O represents an even number, what is the only possible value for W?
Solution
Since both T's are 7, then O has to equal 4, because 7 + 7 = 14. Then, F has to equal 1. To get R, we do 4 + 4 (since O = 4) to get R = 8. The value for W then has to be a number less than 5, otherwise it will change the value of O, and can't be a number that has already been used, like 4 or 1. The only other possibilities are 2 and 3. 2 doesn't work because it makes U = 4, which is what O already equals. So, the only possible value of W is 3
Video Solution
https://www.youtube.com/watch?v=372B3Hyt3bA
~David
See Also
2003 AMC 8 (Problems • Answer Key • Resources) | ||
Preceded by Problem 13 |
Followed by Problem 15 | |
1 • 2 • 3 • 4 • 5 • 6 • 7 • 8 • 9 • 10 • 11 • 12 • 13 • 14 • 15 • 16 • 17 • 18 • 19 • 20 • 21 • 22 • 23 • 24 • 25 | ||
All AJHSME/AMC 8 Problems and Solutions |
The problems on this page are copyrighted by the Mathematical Association of America's American Mathematics Competitions.